Circular reasoning in L'Hopital's rule The 2019 Stack Overflow Developer Survey Results Are In Announcing the arrival of Valued Associate #679: Cesar Manara Planned maintenance scheduled April 17/18, 2019 at 00:00UTC (8:00pm US/Eastern)L'Hospital's Rule Question.Why doesn't L'Hopital's rule work in this case?Finding limits by L'Hospital's RuleApplication of l'hospital rule to exponential functionL'Hopital's Rule helpAvoiding circular logic using L'Hospital's ruleShow $ lim_t to infty -t^xe^-t = 0 $Applying L'Hopital's Rule without hypothesis on numeratorHow to show that $lim_xto infty left(int^x_2 (ln t)^-1 dt right) big/ (x /ln x)=1$?$lim_x to infty e^x - frace^xx+1$ Application of L'Hopital's Rule

Did God make two great lights or did He make the great light two?

How to politely respond to generic emails requesting a PhD/job in my lab? Without wasting too much time

How does ice melt when immersed in water?

What is special about square numbers here?

Create an outline of font

Do working physicists consider Newtonian mechanics to be "falsified"?

Why did all the guest students take carriages to the Yule Ball?

How can I define good in a religion that claims no moral authority?

What was the last x86 CPU that did not have the x87 floating-point unit built in?

Is above average number of years spent on PhD considered a red flag in future academia or industry positions?

Why can't devices on different VLANs, but on the same subnet, communicate?

How to remove this toilet supply line that seems to have no nut?

How should I replace vector<uint8_t>::const_iterator in an API?

Semisimplicity of the category of coherent sheaves?

Can undead you have reanimated wait inside a portable hole?

In horse breeding, what is the female equivalent of putting a horse out "to stud"?

Segmentation fault output is suppressed when piping stdin into a function. Why?

How does this infinite series simplify to an integral?

Netflix Recommendations?

The following signatures were invalid: EXPKEYSIG 1397BC53640DB551

How can I protect witches in combat who wear limited clothing?

"... to apply for a visa" or "... and applied for a visa"?

How to delete random line from file using Unix command?

University's motivation for having tenure-track positions



Circular reasoning in L'Hopital's rule



The 2019 Stack Overflow Developer Survey Results Are In
Announcing the arrival of Valued Associate #679: Cesar Manara
Planned maintenance scheduled April 17/18, 2019 at 00:00UTC (8:00pm US/Eastern)L'Hospital's Rule Question.Why doesn't L'Hopital's rule work in this case?Finding limits by L'Hospital's RuleApplication of l'hospital rule to exponential functionL'Hopital's Rule helpAvoiding circular logic using L'Hospital's ruleShow $ lim_t to infty -t^xe^-t = 0 $Applying L'Hopital's Rule without hypothesis on numeratorHow to show that $lim_xto infty left(int^x_2 (ln t)^-1 dt right) big/ (x /ln x)=1$?$lim_x to infty e^x - frace^xx+1$ Application of L'Hopital's Rule










7












$begingroup$


Suppose we have a function $f(x)$ that satisfies:
$$lim_xtoinftyf(x)=L$$
Where $LinmathbbR$. Is this true?
$$lim_xtoinftyf'(x)=0$$



My approach was simply this:



$$lim_xtoinftyf(x)=lim_xtoinftyfracxf(x)x=L$$



And applying L'Hospital's rule we have:



$$lim_xtoinftyfracxf(x)x=lim_xtoinftyfracf(x)+xf'(x)1=L$$
$$lim_xtoinftyf(x)+xf'(x)=L+lim_xtoinftyxf'(x)=L$$
And finally:
$$lim_xtoinftyxf'(x)=0$$
Now, the only way this is possible is if $lim_xtoinftyf'(x)neqinfty$ and $lim_xtoinftyf'(x)neq AinmathbbR$ , because otherways the $lim_xtoinftyxf'(x)$ would go to infinity. In conclusion, $lim_xtoinftyf'(x)=0$



Is this in any way circular reasoning? I'm especially worried about the part when we apply the L'Hospital's rule.










share|cite|improve this question











$endgroup$







  • 3




    $begingroup$
    Something like $f(x)=sin(x^2)/x$ provides a counterexample, doesn't it?
    $endgroup$
    – John Doe
    yesterday











  • $begingroup$
    To use the rule you'd need $xf$, like $x$, to diverge; but in the counterexample others have discussed herein, $xf=sin x^2$ has no $xtoinfty$ limit. If $L$ were nonzero, on the other hand...
    $endgroup$
    – J.G.
    yesterday






  • 2




    $begingroup$
    Application of LHR does not require that the numerator approach ∞ ∞ . In fact, the limit of the numerator need not even exist. What IS required is that the limit of the quotient of derivatives DOES exist. In the counter examples given on this page, that limit fails to exist and therefore invalidates application of LHR.
    $endgroup$
    – Mark Viola
    yesterday










  • $begingroup$
    @MarkViola: On the other hand, then you do need the denominator to approach $infty$. Otherwise you get into trouble with cases like $limlimits_xtoinfty frac2-1/x1-1/x$.
    $endgroup$
    – Henning Makholm
    yesterday











  • $begingroup$
    @HenningMakholm Indeed. The limit of the denominator must approach $infyt$ (or $-infty$).
    $endgroup$
    – Mark Viola
    yesterday















7












$begingroup$


Suppose we have a function $f(x)$ that satisfies:
$$lim_xtoinftyf(x)=L$$
Where $LinmathbbR$. Is this true?
$$lim_xtoinftyf'(x)=0$$



My approach was simply this:



$$lim_xtoinftyf(x)=lim_xtoinftyfracxf(x)x=L$$



And applying L'Hospital's rule we have:



$$lim_xtoinftyfracxf(x)x=lim_xtoinftyfracf(x)+xf'(x)1=L$$
$$lim_xtoinftyf(x)+xf'(x)=L+lim_xtoinftyxf'(x)=L$$
And finally:
$$lim_xtoinftyxf'(x)=0$$
Now, the only way this is possible is if $lim_xtoinftyf'(x)neqinfty$ and $lim_xtoinftyf'(x)neq AinmathbbR$ , because otherways the $lim_xtoinftyxf'(x)$ would go to infinity. In conclusion, $lim_xtoinftyf'(x)=0$



Is this in any way circular reasoning? I'm especially worried about the part when we apply the L'Hospital's rule.










share|cite|improve this question











$endgroup$







  • 3




    $begingroup$
    Something like $f(x)=sin(x^2)/x$ provides a counterexample, doesn't it?
    $endgroup$
    – John Doe
    yesterday











  • $begingroup$
    To use the rule you'd need $xf$, like $x$, to diverge; but in the counterexample others have discussed herein, $xf=sin x^2$ has no $xtoinfty$ limit. If $L$ were nonzero, on the other hand...
    $endgroup$
    – J.G.
    yesterday






  • 2




    $begingroup$
    Application of LHR does not require that the numerator approach ∞ ∞ . In fact, the limit of the numerator need not even exist. What IS required is that the limit of the quotient of derivatives DOES exist. In the counter examples given on this page, that limit fails to exist and therefore invalidates application of LHR.
    $endgroup$
    – Mark Viola
    yesterday










  • $begingroup$
    @MarkViola: On the other hand, then you do need the denominator to approach $infty$. Otherwise you get into trouble with cases like $limlimits_xtoinfty frac2-1/x1-1/x$.
    $endgroup$
    – Henning Makholm
    yesterday











  • $begingroup$
    @HenningMakholm Indeed. The limit of the denominator must approach $infyt$ (or $-infty$).
    $endgroup$
    – Mark Viola
    yesterday













7












7








7


4



$begingroup$


Suppose we have a function $f(x)$ that satisfies:
$$lim_xtoinftyf(x)=L$$
Where $LinmathbbR$. Is this true?
$$lim_xtoinftyf'(x)=0$$



My approach was simply this:



$$lim_xtoinftyf(x)=lim_xtoinftyfracxf(x)x=L$$



And applying L'Hospital's rule we have:



$$lim_xtoinftyfracxf(x)x=lim_xtoinftyfracf(x)+xf'(x)1=L$$
$$lim_xtoinftyf(x)+xf'(x)=L+lim_xtoinftyxf'(x)=L$$
And finally:
$$lim_xtoinftyxf'(x)=0$$
Now, the only way this is possible is if $lim_xtoinftyf'(x)neqinfty$ and $lim_xtoinftyf'(x)neq AinmathbbR$ , because otherways the $lim_xtoinftyxf'(x)$ would go to infinity. In conclusion, $lim_xtoinftyf'(x)=0$



Is this in any way circular reasoning? I'm especially worried about the part when we apply the L'Hospital's rule.










share|cite|improve this question











$endgroup$




Suppose we have a function $f(x)$ that satisfies:
$$lim_xtoinftyf(x)=L$$
Where $LinmathbbR$. Is this true?
$$lim_xtoinftyf'(x)=0$$



My approach was simply this:



$$lim_xtoinftyf(x)=lim_xtoinftyfracxf(x)x=L$$



And applying L'Hospital's rule we have:



$$lim_xtoinftyfracxf(x)x=lim_xtoinftyfracf(x)+xf'(x)1=L$$
$$lim_xtoinftyf(x)+xf'(x)=L+lim_xtoinftyxf'(x)=L$$
And finally:
$$lim_xtoinftyxf'(x)=0$$
Now, the only way this is possible is if $lim_xtoinftyf'(x)neqinfty$ and $lim_xtoinftyf'(x)neq AinmathbbR$ , because otherways the $lim_xtoinftyxf'(x)$ would go to infinity. In conclusion, $lim_xtoinftyf'(x)=0$



Is this in any way circular reasoning? I'm especially worried about the part when we apply the L'Hospital's rule.







limits






share|cite|improve this question















share|cite|improve this question













share|cite|improve this question




share|cite|improve this question








edited yesterday







marcozz

















asked yesterday









marcozzmarcozz

137110




137110







  • 3




    $begingroup$
    Something like $f(x)=sin(x^2)/x$ provides a counterexample, doesn't it?
    $endgroup$
    – John Doe
    yesterday











  • $begingroup$
    To use the rule you'd need $xf$, like $x$, to diverge; but in the counterexample others have discussed herein, $xf=sin x^2$ has no $xtoinfty$ limit. If $L$ were nonzero, on the other hand...
    $endgroup$
    – J.G.
    yesterday






  • 2




    $begingroup$
    Application of LHR does not require that the numerator approach ∞ ∞ . In fact, the limit of the numerator need not even exist. What IS required is that the limit of the quotient of derivatives DOES exist. In the counter examples given on this page, that limit fails to exist and therefore invalidates application of LHR.
    $endgroup$
    – Mark Viola
    yesterday










  • $begingroup$
    @MarkViola: On the other hand, then you do need the denominator to approach $infty$. Otherwise you get into trouble with cases like $limlimits_xtoinfty frac2-1/x1-1/x$.
    $endgroup$
    – Henning Makholm
    yesterday











  • $begingroup$
    @HenningMakholm Indeed. The limit of the denominator must approach $infyt$ (or $-infty$).
    $endgroup$
    – Mark Viola
    yesterday












  • 3




    $begingroup$
    Something like $f(x)=sin(x^2)/x$ provides a counterexample, doesn't it?
    $endgroup$
    – John Doe
    yesterday











  • $begingroup$
    To use the rule you'd need $xf$, like $x$, to diverge; but in the counterexample others have discussed herein, $xf=sin x^2$ has no $xtoinfty$ limit. If $L$ were nonzero, on the other hand...
    $endgroup$
    – J.G.
    yesterday






  • 2




    $begingroup$
    Application of LHR does not require that the numerator approach ∞ ∞ . In fact, the limit of the numerator need not even exist. What IS required is that the limit of the quotient of derivatives DOES exist. In the counter examples given on this page, that limit fails to exist and therefore invalidates application of LHR.
    $endgroup$
    – Mark Viola
    yesterday










  • $begingroup$
    @MarkViola: On the other hand, then you do need the denominator to approach $infty$. Otherwise you get into trouble with cases like $limlimits_xtoinfty frac2-1/x1-1/x$.
    $endgroup$
    – Henning Makholm
    yesterday











  • $begingroup$
    @HenningMakholm Indeed. The limit of the denominator must approach $infyt$ (or $-infty$).
    $endgroup$
    – Mark Viola
    yesterday







3




3




$begingroup$
Something like $f(x)=sin(x^2)/x$ provides a counterexample, doesn't it?
$endgroup$
– John Doe
yesterday





$begingroup$
Something like $f(x)=sin(x^2)/x$ provides a counterexample, doesn't it?
$endgroup$
– John Doe
yesterday













$begingroup$
To use the rule you'd need $xf$, like $x$, to diverge; but in the counterexample others have discussed herein, $xf=sin x^2$ has no $xtoinfty$ limit. If $L$ were nonzero, on the other hand...
$endgroup$
– J.G.
yesterday




$begingroup$
To use the rule you'd need $xf$, like $x$, to diverge; but in the counterexample others have discussed herein, $xf=sin x^2$ has no $xtoinfty$ limit. If $L$ were nonzero, on the other hand...
$endgroup$
– J.G.
yesterday




2




2




$begingroup$
Application of LHR does not require that the numerator approach ∞ ∞ . In fact, the limit of the numerator need not even exist. What IS required is that the limit of the quotient of derivatives DOES exist. In the counter examples given on this page, that limit fails to exist and therefore invalidates application of LHR.
$endgroup$
– Mark Viola
yesterday




$begingroup$
Application of LHR does not require that the numerator approach ∞ ∞ . In fact, the limit of the numerator need not even exist. What IS required is that the limit of the quotient of derivatives DOES exist. In the counter examples given on this page, that limit fails to exist and therefore invalidates application of LHR.
$endgroup$
– Mark Viola
yesterday












$begingroup$
@MarkViola: On the other hand, then you do need the denominator to approach $infty$. Otherwise you get into trouble with cases like $limlimits_xtoinfty frac2-1/x1-1/x$.
$endgroup$
– Henning Makholm
yesterday





$begingroup$
@MarkViola: On the other hand, then you do need the denominator to approach $infty$. Otherwise you get into trouble with cases like $limlimits_xtoinfty frac2-1/x1-1/x$.
$endgroup$
– Henning Makholm
yesterday













$begingroup$
@HenningMakholm Indeed. The limit of the denominator must approach $infyt$ (or $-infty$).
$endgroup$
– Mark Viola
yesterday




$begingroup$
@HenningMakholm Indeed. The limit of the denominator must approach $infyt$ (or $-infty$).
$endgroup$
– Mark Viola
yesterday










2 Answers
2






active

oldest

votes


















15












$begingroup$

Suppose that $f(x)=dfracsin(x^2)x$. Then $lim_xtoinftyf(x)=0$, but the limit $lim_xtoinftyf'(x)$ doesn't exist.



If you try to apply L'Hopital's Rule here as you did, you will be working with$$lim_xtoinftyfracxsin(x^2)x^2.$$But if $g(x)=xsin(x^2)$, then the limit $lim_xtoinftyg'(x)$ doesn't exist. Therefore, you cannot apply L'Hopital's Rule here.






share|cite|improve this answer









$endgroup$








  • 10




    $begingroup$
    so the only thing we can conclude is: if $lim_x to infty f'(x)$ exists, then it must be $0$?
    $endgroup$
    – antkam
    yesterday







  • 2




    $begingroup$
    @antkam Yes, that is correct.
    $endgroup$
    – José Carlos Santos
    yesterday










  • $begingroup$
    Application of LHR does not require that the numerator approach ∞ ∞ . In fact, the limit of the numerator need not even exist. What IS required is that the limit of the quotient of derivatives DOES exist. In the counter examples given on this page, that limit fails to exist and therefore invalidates application of LHR.
    $endgroup$
    – Mark Viola
    yesterday






  • 1




    $begingroup$
    @marcozz: I must say that it's rather disturbing that you are being taught L'Hopital's rule without also the precise conditions under which it holds. This kind of bad pedagogy is the same reason for the fallacy $1 = sqrt1 = sqrt-1·-1 = sqrt-1·sqrt-1 = i·i = -1$. If all rules were taught properly, such fallacies would never even arise.
    $endgroup$
    – user21820
    22 hours ago


















3












$begingroup$

(Paraphrased from Wikipedia.)



L'Hôpital's rule:



Given functions $f$ and $g$ which are differentiable on an open interval $I$, except possibly at a point $c in I$, if



$$ lim _x to cF(x)=lim _xto cG(x)=0 text or pm infty, tag1. $$
$$ G'(x)neq 0 text for all x in I, text with x ne c, text and tag2. $$
$$ lim_x to cfracF'(x)G'(x) text exists. tag3. $$



then



$$lim_x to c fracF(x)G(x) =lim_x to c fracF'(x)G'(x). tag4.$$



You used $F(x) = xf(x)$ and $G(x) = x$ and $I = (x_0, infty)$ for some $x_0 < 0$.



Since $lim _xto inftyG(x)= infty$, condition $(1.)$ requires that
$$lim _x to inftyxf(x) = infty. tagA.$$



Condition $(2.)$ is satisfied by $G(x)=x$.



Condition $(3.)$ requires that
$$lim_x to infty[f(x)+xf'(x)] text exists. tagB.$$



If conditions $(A.)$ and $(B.)$ are met, then, by L'Hôpital's rule,
$$ lim_x to infty f(x) = lim_x to infty[f(x)+xf'(x)]$$



Others have shown you that counter examples do exists.






share|cite|improve this answer









$endgroup$












  • $begingroup$
    Application of LHR does not require that the numerator approach $infty$. In fact, the limit of the numerator need not even exist. What IS required is that the limit of the quotient of derivatives DOES exist. In the counter examples given on this page, that limit fails to exist and therefore invalidates application of LHR. This answer makes that fact explicit.
    $endgroup$
    – Mark Viola
    yesterday







  • 1




    $begingroup$
    @MarkViola Assuming he wants to use LHR on $(xf)(x)$, then since $lim_limitsx to infty G(x) = infty$, then condition 1 requires ...
    $endgroup$
    – steven gregory
    yesterday











  • $begingroup$
    First, I don't presume that the OP is a male. My point is that in "Condition $1$ can be relaxed. It is NOT required that $lim F=infty$.
    $endgroup$
    – Mark Viola
    yesterday











Your Answer








StackExchange.ready(function()
var channelOptions =
tags: "".split(" "),
id: "69"
;
initTagRenderer("".split(" "), "".split(" "), channelOptions);

StackExchange.using("externalEditor", function()
// Have to fire editor after snippets, if snippets enabled
if (StackExchange.settings.snippets.snippetsEnabled)
StackExchange.using("snippets", function()
createEditor();
);

else
createEditor();

);

function createEditor()
StackExchange.prepareEditor(
heartbeatType: 'answer',
autoActivateHeartbeat: false,
convertImagesToLinks: true,
noModals: true,
showLowRepImageUploadWarning: true,
reputationToPostImages: 10,
bindNavPrevention: true,
postfix: "",
imageUploader:
brandingHtml: "Powered by u003ca class="icon-imgur-white" href="https://imgur.com/"u003eu003c/au003e",
contentPolicyHtml: "User contributions licensed under u003ca href="https://creativecommons.org/licenses/by-sa/3.0/"u003ecc by-sa 3.0 with attribution requiredu003c/au003e u003ca href="https://stackoverflow.com/legal/content-policy"u003e(content policy)u003c/au003e",
allowUrls: true
,
noCode: true, onDemand: true,
discardSelector: ".discard-answer"
,immediatelyShowMarkdownHelp:true
);



);













draft saved

draft discarded


















StackExchange.ready(
function ()
StackExchange.openid.initPostLogin('.new-post-login', 'https%3a%2f%2fmath.stackexchange.com%2fquestions%2f3185074%2fcircular-reasoning-in-lhopitals-rule%23new-answer', 'question_page');

);

Post as a guest















Required, but never shown

























2 Answers
2






active

oldest

votes








2 Answers
2






active

oldest

votes









active

oldest

votes






active

oldest

votes









15












$begingroup$

Suppose that $f(x)=dfracsin(x^2)x$. Then $lim_xtoinftyf(x)=0$, but the limit $lim_xtoinftyf'(x)$ doesn't exist.



If you try to apply L'Hopital's Rule here as you did, you will be working with$$lim_xtoinftyfracxsin(x^2)x^2.$$But if $g(x)=xsin(x^2)$, then the limit $lim_xtoinftyg'(x)$ doesn't exist. Therefore, you cannot apply L'Hopital's Rule here.






share|cite|improve this answer









$endgroup$








  • 10




    $begingroup$
    so the only thing we can conclude is: if $lim_x to infty f'(x)$ exists, then it must be $0$?
    $endgroup$
    – antkam
    yesterday







  • 2




    $begingroup$
    @antkam Yes, that is correct.
    $endgroup$
    – José Carlos Santos
    yesterday










  • $begingroup$
    Application of LHR does not require that the numerator approach ∞ ∞ . In fact, the limit of the numerator need not even exist. What IS required is that the limit of the quotient of derivatives DOES exist. In the counter examples given on this page, that limit fails to exist and therefore invalidates application of LHR.
    $endgroup$
    – Mark Viola
    yesterday






  • 1




    $begingroup$
    @marcozz: I must say that it's rather disturbing that you are being taught L'Hopital's rule without also the precise conditions under which it holds. This kind of bad pedagogy is the same reason for the fallacy $1 = sqrt1 = sqrt-1·-1 = sqrt-1·sqrt-1 = i·i = -1$. If all rules were taught properly, such fallacies would never even arise.
    $endgroup$
    – user21820
    22 hours ago















15












$begingroup$

Suppose that $f(x)=dfracsin(x^2)x$. Then $lim_xtoinftyf(x)=0$, but the limit $lim_xtoinftyf'(x)$ doesn't exist.



If you try to apply L'Hopital's Rule here as you did, you will be working with$$lim_xtoinftyfracxsin(x^2)x^2.$$But if $g(x)=xsin(x^2)$, then the limit $lim_xtoinftyg'(x)$ doesn't exist. Therefore, you cannot apply L'Hopital's Rule here.






share|cite|improve this answer









$endgroup$








  • 10




    $begingroup$
    so the only thing we can conclude is: if $lim_x to infty f'(x)$ exists, then it must be $0$?
    $endgroup$
    – antkam
    yesterday







  • 2




    $begingroup$
    @antkam Yes, that is correct.
    $endgroup$
    – José Carlos Santos
    yesterday










  • $begingroup$
    Application of LHR does not require that the numerator approach ∞ ∞ . In fact, the limit of the numerator need not even exist. What IS required is that the limit of the quotient of derivatives DOES exist. In the counter examples given on this page, that limit fails to exist and therefore invalidates application of LHR.
    $endgroup$
    – Mark Viola
    yesterday






  • 1




    $begingroup$
    @marcozz: I must say that it's rather disturbing that you are being taught L'Hopital's rule without also the precise conditions under which it holds. This kind of bad pedagogy is the same reason for the fallacy $1 = sqrt1 = sqrt-1·-1 = sqrt-1·sqrt-1 = i·i = -1$. If all rules were taught properly, such fallacies would never even arise.
    $endgroup$
    – user21820
    22 hours ago













15












15








15





$begingroup$

Suppose that $f(x)=dfracsin(x^2)x$. Then $lim_xtoinftyf(x)=0$, but the limit $lim_xtoinftyf'(x)$ doesn't exist.



If you try to apply L'Hopital's Rule here as you did, you will be working with$$lim_xtoinftyfracxsin(x^2)x^2.$$But if $g(x)=xsin(x^2)$, then the limit $lim_xtoinftyg'(x)$ doesn't exist. Therefore, you cannot apply L'Hopital's Rule here.






share|cite|improve this answer









$endgroup$



Suppose that $f(x)=dfracsin(x^2)x$. Then $lim_xtoinftyf(x)=0$, but the limit $lim_xtoinftyf'(x)$ doesn't exist.



If you try to apply L'Hopital's Rule here as you did, you will be working with$$lim_xtoinftyfracxsin(x^2)x^2.$$But if $g(x)=xsin(x^2)$, then the limit $lim_xtoinftyg'(x)$ doesn't exist. Therefore, you cannot apply L'Hopital's Rule here.







share|cite|improve this answer












share|cite|improve this answer



share|cite|improve this answer










answered yesterday









José Carlos SantosJosé Carlos Santos

174k23134243




174k23134243







  • 10




    $begingroup$
    so the only thing we can conclude is: if $lim_x to infty f'(x)$ exists, then it must be $0$?
    $endgroup$
    – antkam
    yesterday







  • 2




    $begingroup$
    @antkam Yes, that is correct.
    $endgroup$
    – José Carlos Santos
    yesterday










  • $begingroup$
    Application of LHR does not require that the numerator approach ∞ ∞ . In fact, the limit of the numerator need not even exist. What IS required is that the limit of the quotient of derivatives DOES exist. In the counter examples given on this page, that limit fails to exist and therefore invalidates application of LHR.
    $endgroup$
    – Mark Viola
    yesterday






  • 1




    $begingroup$
    @marcozz: I must say that it's rather disturbing that you are being taught L'Hopital's rule without also the precise conditions under which it holds. This kind of bad pedagogy is the same reason for the fallacy $1 = sqrt1 = sqrt-1·-1 = sqrt-1·sqrt-1 = i·i = -1$. If all rules were taught properly, such fallacies would never even arise.
    $endgroup$
    – user21820
    22 hours ago












  • 10




    $begingroup$
    so the only thing we can conclude is: if $lim_x to infty f'(x)$ exists, then it must be $0$?
    $endgroup$
    – antkam
    yesterday







  • 2




    $begingroup$
    @antkam Yes, that is correct.
    $endgroup$
    – José Carlos Santos
    yesterday










  • $begingroup$
    Application of LHR does not require that the numerator approach ∞ ∞ . In fact, the limit of the numerator need not even exist. What IS required is that the limit of the quotient of derivatives DOES exist. In the counter examples given on this page, that limit fails to exist and therefore invalidates application of LHR.
    $endgroup$
    – Mark Viola
    yesterday






  • 1




    $begingroup$
    @marcozz: I must say that it's rather disturbing that you are being taught L'Hopital's rule without also the precise conditions under which it holds. This kind of bad pedagogy is the same reason for the fallacy $1 = sqrt1 = sqrt-1·-1 = sqrt-1·sqrt-1 = i·i = -1$. If all rules were taught properly, such fallacies would never even arise.
    $endgroup$
    – user21820
    22 hours ago







10




10




$begingroup$
so the only thing we can conclude is: if $lim_x to infty f'(x)$ exists, then it must be $0$?
$endgroup$
– antkam
yesterday





$begingroup$
so the only thing we can conclude is: if $lim_x to infty f'(x)$ exists, then it must be $0$?
$endgroup$
– antkam
yesterday





2




2




$begingroup$
@antkam Yes, that is correct.
$endgroup$
– José Carlos Santos
yesterday




$begingroup$
@antkam Yes, that is correct.
$endgroup$
– José Carlos Santos
yesterday












$begingroup$
Application of LHR does not require that the numerator approach ∞ ∞ . In fact, the limit of the numerator need not even exist. What IS required is that the limit of the quotient of derivatives DOES exist. In the counter examples given on this page, that limit fails to exist and therefore invalidates application of LHR.
$endgroup$
– Mark Viola
yesterday




$begingroup$
Application of LHR does not require that the numerator approach ∞ ∞ . In fact, the limit of the numerator need not even exist. What IS required is that the limit of the quotient of derivatives DOES exist. In the counter examples given on this page, that limit fails to exist and therefore invalidates application of LHR.
$endgroup$
– Mark Viola
yesterday




1




1




$begingroup$
@marcozz: I must say that it's rather disturbing that you are being taught L'Hopital's rule without also the precise conditions under which it holds. This kind of bad pedagogy is the same reason for the fallacy $1 = sqrt1 = sqrt-1·-1 = sqrt-1·sqrt-1 = i·i = -1$. If all rules were taught properly, such fallacies would never even arise.
$endgroup$
– user21820
22 hours ago




$begingroup$
@marcozz: I must say that it's rather disturbing that you are being taught L'Hopital's rule without also the precise conditions under which it holds. This kind of bad pedagogy is the same reason for the fallacy $1 = sqrt1 = sqrt-1·-1 = sqrt-1·sqrt-1 = i·i = -1$. If all rules were taught properly, such fallacies would never even arise.
$endgroup$
– user21820
22 hours ago











3












$begingroup$

(Paraphrased from Wikipedia.)



L'Hôpital's rule:



Given functions $f$ and $g$ which are differentiable on an open interval $I$, except possibly at a point $c in I$, if



$$ lim _x to cF(x)=lim _xto cG(x)=0 text or pm infty, tag1. $$
$$ G'(x)neq 0 text for all x in I, text with x ne c, text and tag2. $$
$$ lim_x to cfracF'(x)G'(x) text exists. tag3. $$



then



$$lim_x to c fracF(x)G(x) =lim_x to c fracF'(x)G'(x). tag4.$$



You used $F(x) = xf(x)$ and $G(x) = x$ and $I = (x_0, infty)$ for some $x_0 < 0$.



Since $lim _xto inftyG(x)= infty$, condition $(1.)$ requires that
$$lim _x to inftyxf(x) = infty. tagA.$$



Condition $(2.)$ is satisfied by $G(x)=x$.



Condition $(3.)$ requires that
$$lim_x to infty[f(x)+xf'(x)] text exists. tagB.$$



If conditions $(A.)$ and $(B.)$ are met, then, by L'Hôpital's rule,
$$ lim_x to infty f(x) = lim_x to infty[f(x)+xf'(x)]$$



Others have shown you that counter examples do exists.






share|cite|improve this answer









$endgroup$












  • $begingroup$
    Application of LHR does not require that the numerator approach $infty$. In fact, the limit of the numerator need not even exist. What IS required is that the limit of the quotient of derivatives DOES exist. In the counter examples given on this page, that limit fails to exist and therefore invalidates application of LHR. This answer makes that fact explicit.
    $endgroup$
    – Mark Viola
    yesterday







  • 1




    $begingroup$
    @MarkViola Assuming he wants to use LHR on $(xf)(x)$, then since $lim_limitsx to infty G(x) = infty$, then condition 1 requires ...
    $endgroup$
    – steven gregory
    yesterday











  • $begingroup$
    First, I don't presume that the OP is a male. My point is that in "Condition $1$ can be relaxed. It is NOT required that $lim F=infty$.
    $endgroup$
    – Mark Viola
    yesterday















3












$begingroup$

(Paraphrased from Wikipedia.)



L'Hôpital's rule:



Given functions $f$ and $g$ which are differentiable on an open interval $I$, except possibly at a point $c in I$, if



$$ lim _x to cF(x)=lim _xto cG(x)=0 text or pm infty, tag1. $$
$$ G'(x)neq 0 text for all x in I, text with x ne c, text and tag2. $$
$$ lim_x to cfracF'(x)G'(x) text exists. tag3. $$



then



$$lim_x to c fracF(x)G(x) =lim_x to c fracF'(x)G'(x). tag4.$$



You used $F(x) = xf(x)$ and $G(x) = x$ and $I = (x_0, infty)$ for some $x_0 < 0$.



Since $lim _xto inftyG(x)= infty$, condition $(1.)$ requires that
$$lim _x to inftyxf(x) = infty. tagA.$$



Condition $(2.)$ is satisfied by $G(x)=x$.



Condition $(3.)$ requires that
$$lim_x to infty[f(x)+xf'(x)] text exists. tagB.$$



If conditions $(A.)$ and $(B.)$ are met, then, by L'Hôpital's rule,
$$ lim_x to infty f(x) = lim_x to infty[f(x)+xf'(x)]$$



Others have shown you that counter examples do exists.






share|cite|improve this answer









$endgroup$












  • $begingroup$
    Application of LHR does not require that the numerator approach $infty$. In fact, the limit of the numerator need not even exist. What IS required is that the limit of the quotient of derivatives DOES exist. In the counter examples given on this page, that limit fails to exist and therefore invalidates application of LHR. This answer makes that fact explicit.
    $endgroup$
    – Mark Viola
    yesterday







  • 1




    $begingroup$
    @MarkViola Assuming he wants to use LHR on $(xf)(x)$, then since $lim_limitsx to infty G(x) = infty$, then condition 1 requires ...
    $endgroup$
    – steven gregory
    yesterday











  • $begingroup$
    First, I don't presume that the OP is a male. My point is that in "Condition $1$ can be relaxed. It is NOT required that $lim F=infty$.
    $endgroup$
    – Mark Viola
    yesterday













3












3








3





$begingroup$

(Paraphrased from Wikipedia.)



L'Hôpital's rule:



Given functions $f$ and $g$ which are differentiable on an open interval $I$, except possibly at a point $c in I$, if



$$ lim _x to cF(x)=lim _xto cG(x)=0 text or pm infty, tag1. $$
$$ G'(x)neq 0 text for all x in I, text with x ne c, text and tag2. $$
$$ lim_x to cfracF'(x)G'(x) text exists. tag3. $$



then



$$lim_x to c fracF(x)G(x) =lim_x to c fracF'(x)G'(x). tag4.$$



You used $F(x) = xf(x)$ and $G(x) = x$ and $I = (x_0, infty)$ for some $x_0 < 0$.



Since $lim _xto inftyG(x)= infty$, condition $(1.)$ requires that
$$lim _x to inftyxf(x) = infty. tagA.$$



Condition $(2.)$ is satisfied by $G(x)=x$.



Condition $(3.)$ requires that
$$lim_x to infty[f(x)+xf'(x)] text exists. tagB.$$



If conditions $(A.)$ and $(B.)$ are met, then, by L'Hôpital's rule,
$$ lim_x to infty f(x) = lim_x to infty[f(x)+xf'(x)]$$



Others have shown you that counter examples do exists.






share|cite|improve this answer









$endgroup$



(Paraphrased from Wikipedia.)



L'Hôpital's rule:



Given functions $f$ and $g$ which are differentiable on an open interval $I$, except possibly at a point $c in I$, if



$$ lim _x to cF(x)=lim _xto cG(x)=0 text or pm infty, tag1. $$
$$ G'(x)neq 0 text for all x in I, text with x ne c, text and tag2. $$
$$ lim_x to cfracF'(x)G'(x) text exists. tag3. $$



then



$$lim_x to c fracF(x)G(x) =lim_x to c fracF'(x)G'(x). tag4.$$



You used $F(x) = xf(x)$ and $G(x) = x$ and $I = (x_0, infty)$ for some $x_0 < 0$.



Since $lim _xto inftyG(x)= infty$, condition $(1.)$ requires that
$$lim _x to inftyxf(x) = infty. tagA.$$



Condition $(2.)$ is satisfied by $G(x)=x$.



Condition $(3.)$ requires that
$$lim_x to infty[f(x)+xf'(x)] text exists. tagB.$$



If conditions $(A.)$ and $(B.)$ are met, then, by L'Hôpital's rule,
$$ lim_x to infty f(x) = lim_x to infty[f(x)+xf'(x)]$$



Others have shown you that counter examples do exists.







share|cite|improve this answer












share|cite|improve this answer



share|cite|improve this answer










answered yesterday









steven gregorysteven gregory

18.4k32359




18.4k32359











  • $begingroup$
    Application of LHR does not require that the numerator approach $infty$. In fact, the limit of the numerator need not even exist. What IS required is that the limit of the quotient of derivatives DOES exist. In the counter examples given on this page, that limit fails to exist and therefore invalidates application of LHR. This answer makes that fact explicit.
    $endgroup$
    – Mark Viola
    yesterday







  • 1




    $begingroup$
    @MarkViola Assuming he wants to use LHR on $(xf)(x)$, then since $lim_limitsx to infty G(x) = infty$, then condition 1 requires ...
    $endgroup$
    – steven gregory
    yesterday











  • $begingroup$
    First, I don't presume that the OP is a male. My point is that in "Condition $1$ can be relaxed. It is NOT required that $lim F=infty$.
    $endgroup$
    – Mark Viola
    yesterday
















  • $begingroup$
    Application of LHR does not require that the numerator approach $infty$. In fact, the limit of the numerator need not even exist. What IS required is that the limit of the quotient of derivatives DOES exist. In the counter examples given on this page, that limit fails to exist and therefore invalidates application of LHR. This answer makes that fact explicit.
    $endgroup$
    – Mark Viola
    yesterday







  • 1




    $begingroup$
    @MarkViola Assuming he wants to use LHR on $(xf)(x)$, then since $lim_limitsx to infty G(x) = infty$, then condition 1 requires ...
    $endgroup$
    – steven gregory
    yesterday











  • $begingroup$
    First, I don't presume that the OP is a male. My point is that in "Condition $1$ can be relaxed. It is NOT required that $lim F=infty$.
    $endgroup$
    – Mark Viola
    yesterday















$begingroup$
Application of LHR does not require that the numerator approach $infty$. In fact, the limit of the numerator need not even exist. What IS required is that the limit of the quotient of derivatives DOES exist. In the counter examples given on this page, that limit fails to exist and therefore invalidates application of LHR. This answer makes that fact explicit.
$endgroup$
– Mark Viola
yesterday





$begingroup$
Application of LHR does not require that the numerator approach $infty$. In fact, the limit of the numerator need not even exist. What IS required is that the limit of the quotient of derivatives DOES exist. In the counter examples given on this page, that limit fails to exist and therefore invalidates application of LHR. This answer makes that fact explicit.
$endgroup$
– Mark Viola
yesterday





1




1




$begingroup$
@MarkViola Assuming he wants to use LHR on $(xf)(x)$, then since $lim_limitsx to infty G(x) = infty$, then condition 1 requires ...
$endgroup$
– steven gregory
yesterday





$begingroup$
@MarkViola Assuming he wants to use LHR on $(xf)(x)$, then since $lim_limitsx to infty G(x) = infty$, then condition 1 requires ...
$endgroup$
– steven gregory
yesterday













$begingroup$
First, I don't presume that the OP is a male. My point is that in "Condition $1$ can be relaxed. It is NOT required that $lim F=infty$.
$endgroup$
– Mark Viola
yesterday




$begingroup$
First, I don't presume that the OP is a male. My point is that in "Condition $1$ can be relaxed. It is NOT required that $lim F=infty$.
$endgroup$
– Mark Viola
yesterday

















draft saved

draft discarded
















































Thanks for contributing an answer to Mathematics Stack Exchange!


  • Please be sure to answer the question. Provide details and share your research!

But avoid


  • Asking for help, clarification, or responding to other answers.

  • Making statements based on opinion; back them up with references or personal experience.

Use MathJax to format equations. MathJax reference.


To learn more, see our tips on writing great answers.




draft saved


draft discarded














StackExchange.ready(
function ()
StackExchange.openid.initPostLogin('.new-post-login', 'https%3a%2f%2fmath.stackexchange.com%2fquestions%2f3185074%2fcircular-reasoning-in-lhopitals-rule%23new-answer', 'question_page');

);

Post as a guest















Required, but never shown





















































Required, but never shown














Required, but never shown












Required, but never shown







Required, but never shown

































Required, but never shown














Required, but never shown












Required, but never shown







Required, but never shown







-limits

Popular posts from this blog

Mobil Contents History Mobil brands Former Mobil brands Lukoil transaction Mobil UK Mobil Australia Mobil New Zealand Mobil Greece Mobil in Japan Mobil in Canada Mobil Egypt See also References External links Navigation menuwww.mobil.com"Mobil Corporation"the original"Our Houston campus""Business & Finance: Socony-Vacuum Corp.""Popular Mechanics""Lubrite Technologies""Exxon Mobil campus 'clearly happening'""Toledo Blade - Google News Archive Search""The Lion and the Moose - How 2 Executives Pulled off the Biggest Merger Ever""ExxonMobil Press Release""Lubricants""Archived copy"the original"Mobil 1™ and Mobil Super™ motor oil and synthetic motor oil - Mobil™ Motor Oils""Mobil Delvac""Mobil Industrial website""The State of Competition in Gasoline Marketing: The Effects of Refiner Operations at Retail""Mobil Travel Guide to become Forbes Travel Guide""Hotel Rankings: Forbes Merges with Mobil"the original"Jamieson oil industry history""Mobil news""Caltex pumps for control""Watchdog blocks Caltex bid""Exxon Mobil sells service station network""Mobil Oil New Zealand Limited is New Zealand's oldest oil company, with predecessor companies having first established a presence in the country in 1896""ExxonMobil subsidiaries have a business history in New Zealand stretching back more than 120 years. We are involved in petroleum refining and distribution and the marketing of fuels, lubricants and chemical products""Archived copy"the original"Exxon Mobil to Sell Its Japanese Arm for $3.9 Billion""Gas station merger will end Esso and Mobil's long run in Japan""Esso moves to affiliate itself with PC Optimum, no longer Aeroplan, in loyalty point switch""Mobil brand of gas stations to launch in Canada after deal for 213 Loblaws-owned locations""Mobil Nears Completion of Rebranding 200 Loblaw Gas Stations""Learn about ExxonMobil's operations in Egypt""Petrol and Diesel Service Stations in Egypt - Mobil"Official websiteExxon Mobil corporate websiteMobil Industrial official websiteeeeeeeeDA04275022275790-40000 0001 0860 5061n82045453134887257134887257

Frič See also Navigation menuinternal link

Identify plant with long narrow paired leaves and reddish stems Planned maintenance scheduled April 17/18, 2019 at 00:00UTC (8:00pm US/Eastern) Announcing the arrival of Valued Associate #679: Cesar Manara Unicorn Meta Zoo #1: Why another podcast?What is this plant with long sharp leaves? Is it a weed?What is this 3ft high, stalky plant, with mid sized narrow leaves?What is this young shrub with opposite ovate, crenate leaves and reddish stems?What is this plant with large broad serrated leaves?Identify this upright branching weed with long leaves and reddish stemsPlease help me identify this bulbous plant with long, broad leaves and white flowersWhat is this small annual with narrow gray/green leaves and rust colored daisy-type flowers?What is this chilli plant?Does anyone know what type of chilli plant this is?Help identify this plant